Difference between revisions of "2010 AMC 12B Problems/Problem 21"

(Redirected page to 2010 AMC 10B Problems/Problem 25)
(Tag: New redirect)
Line 1: Line 1:
== Problem 21 ==
+
#redirect [[2010 AMC 10B Problems/Problem 25]]
Let <math>a > 0</math>, and let <math>P(x)</math> be a polynomial with integer coefficients such that
 
 
 
<center>
 
<math>P(1) = P(3) = P(5) = P(7) = a</math>, and<br/>
 
<math>P(2) = P(4) = P(6) = P(8) = -a</math>.
 
</center>
 
 
 
What is the smallest possible value of <math>a</math>?
 
 
 
<math>\textbf{(A)}\ 105 \qquad \textbf{(B)}\ 315 \qquad \textbf{(C)}\ 945 \qquad \textbf{(D)}\ 7! \qquad \textbf{(E)}\ 8!</math>
 
 
 
== Solution 1 ==
 
There must be some polynomial <math>Q(x)</math> such that <math>P(x)-a=(x-1)(x-3)(x-5)(x-7)Q(x)</math>
 
 
 
Then, plugging in values of <math>2,4,6,8,</math> we get
 
 
 
<cmath>P(2)-a=(2-1)(2-3)(2-5)(2-7)Q(2) = -15Q(2) = -2a</cmath>
 
<cmath>P(4)-a=(4-1)(4-3)(4-5)(4-7)Q(4) = 9Q(4) = -2a</cmath>
 
<cmath>P(6)-a=(6-1)(6-3)(6-5)(6-7)Q(6) = -15Q(6) = -2a</cmath>
 
<cmath>P(8)-a=(8-1)(8-3)(8-5)(8-7)Q(8) = 105Q(8) = -2a</cmath>
 
<cmath>-2a=-15Q(2)=9Q(4)=-15Q(6)=105Q(8).</cmath>
 
Thus, <math>a</math> must be a multiple of <math>\text{lcm}(15,9,15,105)=315</math>.
 
 
 
Now we show that there exists <math>Q(x)</math> such that <math>a=315.</math> We have
 
<cmath>Q(2)=42, Q(4)=-70, Q(6)=42, Q(8)=-6</cmath>
 
Thus, <math>Q(x)=R(x)(x-2)(x-6)+42</math> for some <math>R(x).</math> From here it is clear that <math>Q(x)</math> exists, since we can take <math>R(x)=-8x+60.</math>
 
 
 
Therefore, our answer is <math> \boxed{\textbf{(B)}\ 315.} </math>
 
 
 
== Solution 2 (Calculus)==
 
The evenly-spaced data suggests using [[discrete derivative|discrete derivatives]] to tackle this problem.  First, note that any polynomial of degree <math>n</math>
 
 
 
<center><math>P(x) = a_0 + a_1 x + a_2 x^2 + \ldots + a_n x^n</math></center>
 
 
 
can also be written as
 
 
 
<center><math>P(x) = b_0 + b_1 (x-1) + b_2 (x-1)(x-2) + \ldots + b_n (x-1)(x-2) \cdots (x-n)</math>.</center>
 
 
 
Moreover, the coefficients <math>a_i</math> are integers for <math>i=0, 1, 2, \ldots n</math> iff the coefficients <math>b_i</math> are integers for <math>i=0, 1, 2, \ldots n</math>.  This latter form is convenient for calculating discrete derivatives of <math>P(x)</math>.
 
 
 
The discrete derivative of a function <math>f(x)</math> is the related function <math>\Delta f(x)</math> defined as
 
 
 
<center><math>\Delta f(x) = f(x+1) - f(x)</math>.</center>
 
 
 
With this definition, it's easy to see that for any positive integer <math>k</math> we have
 
 
 
<center><math>\Delta [(x-1)(x-2)\cdots(x-k)] = k(x-1)(x-2)\cdots(x-[k-1])</math>.</center>
 
 
 
This in turn allows us to use successive discrete derivatives evaluated at <math>x=1</math> to calculate all of the coefficients <math>b_i</math> using
 
 
 
<center><math>P(1)=b_0</math>, <math>\Delta P(1) = b_1</math>, <math>\Delta^2 P(1) = 2 b_2</math>, <math>\ldots</math>, <math>\Delta^7 P(1) = 7! b_7</math>.</center>
 
 
 
We can also calculate the following table of discrete derivatives based on the data points given in the problem statement:
 
 
 
<center>
 
<table frame='box' rules='all' cellpadding='3'>
 
<tr><th /><th colspan='8'><math>x</math></th></tr>
 
 
 
<tr><td align='right'></td><td align='center'><math>1</math></td><td align='center'><math>2</math></td><td align='center'><math>3</math></td><td align='center'><math>4</math></td><td align='center'><math>5</math></td><td align='center'><math>6</math></td><td align='center'><math>7</math></td><td align='center'><math>8</math></td></tr>
 
 
 
<tr><td align='right'><math>P(x)</math></td><td align='right'><math>a</math></td><td align='right'><math>-a</math></td><td align='right'><math>a</math></td><td align='right'><math>-a</math></td><td align='right'><math>a</math></td><td align='right'><math>-a</math></td><td align='right'><math>a</math></td><td align='right'><math>-a</math></td></tr>
 
 
 
<tr><td align='right'><math>\Delta P(x)</math></td><td align='right'><math>-2a</math></td><td align='right'><math>2a</math></td><td align='right'><math>-2a</math></td><td align='right'><math>2a</math></td><td align='right'><math>-2a</math></td><td align='right'><math>2a</math></td><td align='right'><math>-2a</math></td><td /></tr>
 
 
 
<tr><td align='right'><math>\Delta^2 P(x)</math></td><td align='right'><math>4a</math></td><td align='right'><math>-4a</math></td><td align='right'><math>4a</math></td><td align='right'><math>-4a</math></td><td align='right'><math>4a</math></td><td align='right'><math>-4a</math></td><td /><td /></tr>
 
 
 
<tr><td colspan='9' align='center'><math>\vdots</math></td></tr>
 
 
 
<tr><td align='right'><math>\Delta^7 P(x)</math></td><td align='right'><math>-2^7 a</math></td><td /><td /><td /><td /><td /><td /><td /></tr>
 
</table>
 
</center>
 
 
 
Thus we can read down the column for <math>x=1</math> to find that <math>k! b_k = (-2)^k a</math> for <math>k = 0, 1, \ldots, 7</math>.  Interestingly, even if we choose <math>P(x)</math> to have degree greater than <math>7</math>, the <math>8</math> coefficients of lowest order always satisfy these conditions.  Moreover, it's straightforward to show that the <math>P(x)</math> of degree <math>7</math> with <math>b_k</math> satisfying these conditions will fit the data given in the problem statement.  Thus, to find minimal necessary and sufficient conditions on the value of <math>a</math>, we need only consider these <math>8</math> equations.  As a result, <math>P(x)</math> with integer coefficients fitting the given data exists iff <math>k!</math> divides <math>2^k a</math> for <math>k = 0, 1, \ldots, 7</math>.  In other words, it's necessary and sufficient that
 
 
 
<center>
 
<math>0! | a</math>,
 
 
 
<math>1! | 2a</math>,
 
 
 
<math>2! | 2^2 a</math>,
 
 
 
<math>3! | 2^3 a</math>,
 
 
 
<math>4! | 2^4 a</math>,
 
 
 
<math>5! | 2^5 a</math>,
 
 
 
<math>6! | 2^6 a</math>, and
 
 
 
<math>7! | 2^7 a</math>.
 
</center>
 
 
 
The last condition holds iff <math>7 \cdot 3 \cdot 5 \cdot 3 = 315</math> divides evenly into <math>a</math>.  Since such <math>a</math> will also satisfy the first <math>7</math> conditions, our answer is <math> \boxed{\textbf{(B)}\ 315} </math>.
 
 
 
== See also ==
 
{{AMC12 box|year=2010|num-b=20|num-a=22|ab=B}}
 
 
 
[[Category:Intermediate Algebra Problems]]
 
{{MAA Notice}}
 

Revision as of 20:56, 26 May 2020